Anda di halaman 1dari 15

JC1 H2 Maths

Weekend Assignment : Practice Paper for Final Exam (100 marks)

Section A : Pure Maths (40 marks)

1
1. Express f ( x) = in partial fractions.
( x − 1) 2 ( x + 3)
Hence find the expansion of f ( x) , in ascending powers of x, up to and
including the term in x 3 , where x < 1 .
[6]
1 1 1 1 5 22 2 86 3
[Ans : − + + ; + x+ x + x + ... ]
16( x − 1) 4( x − 1) 16( x + 3) 3 9
2
27 81

n
r 2r −1 2n 1
2. Prove by mathematical induction that ∑
r =1 ( r + 1)( r + 2)
= − .
n +2 2
[3]
 r 2r −1 14

Use the above result to evaluate ∑  − r .
r = 7  ( r + 1)( r + 2) 
[3]
[Ans : 932]

n n
n n
3. Given that ∑ ( 2r − 1) 2
r =1 3
2

r =1
2
6

= ( 4n − 1) , show that r = (n + 1)(2n + 1) . Hence find

the sum of the finite sequence 32 , 6 2 , 9 2 , 12 2 , ...., 99 2 .


[6]
[Ans : 112761]

4. n!
It is given that α n = and β n = ( n + k ) αn ,
( n + k) !
where n and k are integers such that n ≥ 1 and k ≥ 2 .

(i) Prove that β n − βn+1 = ( k −1) αn .


[2]

N
1  1 ( N + 1) ! 
(ii) Hence show that ∑α
n =1
n =  − .
k − 1  k ! ( N + k ) !
[3]


(iii) Using your answer in part (ii), find ∑α
n =1
n .

[2]
1
[Ans : (iii) = ]
( k − 1) k !

1
5. The function f is defined as f ( x ) = 4 x − x for x ∈ ¡ .
2

(i) Show that f is not one-one.


[1]
(ii) The domain of f is restricted to x ≤ A such that f −1 exists. State the
maximum value of A and find the range of f .
[2]
(iii) Find the inverse function f −1 .
[3]
(iv) Sketch the graphs of f and f −1 . State the relationship between the
graphs. [2]
[Ans : (ii) 2 (iii) (−∞, 4] ]

6. The ‘Running’ award is given to any student who can complete a 5 km run
in less than half an hour. In order to get the award, Tony trains hard by
running 5 km daily. He takes 45 minutes to complete the 5 km run on the
first day of training, and his timing on each successive day is 0.985 of his
previous day’s timing.
(i) Write down an expression for the time taken for Tony to complete the 5
km run on any particular nth day. Hence find the least number of days
he would need to train to receive the ‘Running’ award.
[3]

As a result of bad weather conditions, doctors recommend a maximum of 15


hours of strenuous outdoor activity in a period of 4 weeks. Those who
exceed 15 hours are said to risk having asthma. Assume that the only form
of strenuous outdoor activity that Tony engages in is the daily 5 km run.

(ii) Will Tony put himself at risk of having asthma if he trains daily until he
obtains the “Running” award? Justify your answer with clearly shown
working. [4]
[Ans : (i) 28 ]

Section B : Statistics (60 marks)

7. (a) Find the number of different arrangements of the eight letters in the
word INFINITY in which

(i) the three I’s are together and the first and last letters are N, [1]

(ii) either the first or last letter (or both) is I.


[3]

(b) Find the number of ways in which 8 students can be arranged in a row,
such that 4 particular students are separated from one another. [2]

Eight students are instead seated at a square table, with 2 seats on


each side. Find the number of ways in which each of the 4 particular
students are seated at 4 different sides of the table given that the seats
are unnumbered. [3]
[Ans : (ai) 24 (aii) 2160 (b) 2880; 2304]

8. Tom and Jerry play a game with a bag containing 20 red balls and 20 green
balls. Tom begins by choosing two balls at random from the bag. If they are

2
both red, Tom wins the game. If not, Jerry chooses two balls at random from
the remaining balls. If they are both red, Jerry wins the game. Otherwise the
game ends in a draw.
Find, to 3 decimal places, the probability that
(a) Tom wins the game.
[2]

(b) Jerry wins the game.


[3]

(c) Jerry wins given that Tom has drawn a red ball and a green ball.
[2]

A series of 4 independent games is played. Find the probability that Tom and
Jerry each wins 2 games, given that all 4 games are won.
[4]
[Ans : (a) 0.244 (b) 0.191 (c) 0.243; 0.364]

9. A fruit retailer bought apples and oranges from the wholesale market. He
weighed the apples and found that 10% of the apples weighed more than
200g and 15% of the apples weighed less than 180g. The mass, in grams, of
a randomly chosen apple may be taken to have a normal distribution N ( µ , σ 2 )
.
Show than µ = 188.94 and σ = 8.63 .
[4]

The mass, in grams, of a randomly chosen orange may also be taken to have
a normal distribution with mean 220 and variance 9. Assuming that the mass
of apples and the mass of oranges are independent, find the probability that
the difference in mass between an apple and an orange is less than 10g.
[3]

The retailer decides to sell the fruits in boxes. There are three apples and two
oranges per box. Given that the mass of the empty box is 52g, find the
probability that the total mass of a randomly chosen box of fruits exceeds
1030g. [3]
[Ans : 0.0106; 0.968]

10. An Air Force analyses the number of technical faults for their two types of
fighter jets V-22 and F-16. It is found that the V-22 has an average number of
6 faults per 100,000 flight hours. The F-16 has an average number of 4 faults
per 100,000 flight hours. It is assumed that the faults occur independently of
one another.
(i) Give a reason to explain why Poisson distribution can be considered to
be a suitable model for the number of faults in a given number of flight
hours. [1]

Assume that the V-22 and F-16 fighter jets each clocks a total of 20,000
hours each year.
(ii) Find the probability that, for a year, there are at least 2 faults for the V-
22 fighter jet and at least 1 fault for F-16 fighter jet, given that there is
a total of 4 faults in that year. [3]
(iii) Find the least value of n such that the probability that there will be a

3
total of more than n faults in a year is not more than 0.15. [2]
[Ans : (ii) 0.691 (iii) 3]

11. A company produces a special type of chocolates called Maltz. The pieces of
Maltz are packed into packets of 16. It is known that 5% of the pieces of Maltz
produced are of poor quality. A packet of Maltz is rejected if it contains at
least 2 pieces that are of poor quality.

(i) Show that the probability that a randomly chosen packet of Maltz is
rejected is 0.189, correct to 3 significant figures.
[2]
(ii) Given that a randomly chosen packet of Maltz is rejected, find the
probability that it contains exactly 2 pieces that are of poor quality.
[2]
(iii) During the festive seasons, the packets of Maltz are packed into boxes
of 50 each. Using a suitable approximation, find the probability that a
randomly chosen box of Maltz will contain at most 10 rejected packets.
[4]
[Ans : (ii) 0.774 (iii) 0.648]

12. A social worker wants to gain statistical evidence that the average amount of
$260 received by public assistance recipients is not enough. The following
observations were obtained when she interviewed 15 recipients on how much
they need to spend per month:
$246, $237, $260, $261, $235, $270, $267, $261, $255, $290, $263, $262,
$291, $259, $298
(i) Set up the appropriate hypotheses. Use the p-value to determine the
level of significance if the amount of money received by public
assistance recipients is enough. State any assumption(s) that you need
to make. [4]

(ii) She decides to interview a large number of recipients and conduct a


different test at 1% level of significance. Stating any assumption(s)
necessary for your test, find the least number of recipients she should
interview in order to make her evidence credible.

[4]
[Ans : (i) 22.4 % level of significance (ii) 134]

13. A PE teacher wants to determine if there is any correlation between the


weight of and the mean amount of money spent on canteen food per month
by an 18-year-old college student. A group of eight 18-year-old college
students was interviewed and had their weights taken. The results are
tabulated as follow:

Weight, x / kg 34 57 43 65 72 46 51 54
Mean amount spent
on food per month, 21.6 58.1 32.8 59.3 60.6 k 56.8 49.5
y/$

(i) Given that the equation of the least squares regression line of y on x
is y = 1.0031x − 4.3018 , find the value of k, leaving your answer in 1
decimal place. [3]

4
(ii) Calculate the value of the linear product moment correlation
coefficient for these eight college students.
[1] [1]
(iii) The following models for the above data are suggested for b > 0:
b
(A) y = a + bx ; (B) y = a + bx 2 ; (C) y = a + b ln x ; (D) y = a +
x
.
State, with a reason, which model is most appropriate.
[2]

(iv) With the choice of the model indicated in (iii), calculate the least
squares estimates of a and b and calculate the value of the linear
product moment correlation coefficient for the transformed data.
[2]

(v) Suppose we have an 18-year-old college student who spends an


average of $80 per month on food, determine the weight of this
student using the least squares regression line obtained in (iv).
Comment on the suitability of your answer. [2]
[Ans : (i) 50.2 (ii) 0.863 (iii) Model (C) (iv) a = −163.14, b = 53.72, r = 0.905 (v)
92.39]

The End

Solutions :

Pure Maths

1. 1 A B C
f ( x) = = + +
( x − 1) ( x + 3) x − 1 ( x − 1)
2 2
x+3
A( x − 1)( x + 3) + B ( x + 3) + C ( x − 1)2
=
( x − 1)2 ( x + 3)
1 = A( x − 1)( x + 3) + B ( x + 3) + C ( x − 1)2
1
By substituting x = –3, C = .
16
1
Comparing coefficients of x 2 , A + C = 0 ⇒ A= − .
16
1
By substituting x = 1, B = .
4
1 1 1 1
f ( x) = =− + +
( x − 1) ( x + 3)
2
16( x − 1) 4( x − 1) 16( x + 3)
2

1 1 1 x
= (1− x) −1 + (1− x) − 2 + (3− 1)(1+ )− 1
16 4 16 3
1 1 1 x
= (1 − x) −1 + (1− x)−2 + (1+ )−1
16 4 48 3

5
1
= (1 + x + x 2 + x 3 + ...) +
16
1
(1 + 2 x + 3x 2 + 4 x 3 + ...) +
4
1 x x2 x3
(1 − + − + ...)
48 3 9 27
1 5 22 2 86 3
= + x+ x + x + ...
3 9 27 81

2. n
r 2r −1 2n 1
Let P ( n ) be the statement ∑ = − .
r =1 ( r + 1)( r + 2) n+ 2 2
1 1 3 1 1
When n = 1, LHS = = ; RHS = − = .
2⋅3 6 2 2 6
∴ LHS = RHS. Hence, P( 1) is true.

Assume P ( k ) is true for some positive integer k, i.e.


k
r 2r −1 2k 1

r =1 ( r + 1)(r + 2)
= − .
k +2 2
k +1
r 2r −1 2k +1 1
Need to prove that P ( k + 1) is true, i.e. ∑ = − .
r =1 ( r + 1)( r + 2) k +3 2
LHS :
k +1
r 2r −1
∑ (r + 1)(r + 2)
r =1
k
r 2r −1
=∑ + Tk +1
r =1 ( r + 1)( r + 2)

2k 1 ( k +1)2k
= − +
k + 2 2 (k + 2)(k + 3)
2k  k + 1  1
= 1+ −
k + 2  k + 3 2
2k  2 k + 4  1
=  −
k +2 k+3  2
2k +1 1
= − = RHS
k +3 2

6
14
r 2r −1 14

∑ (r + 1)(r + 2) − ∑ r
r =7 r =7
r −1
14
r2 6
r 2r −1 14
=∑ −∑ −∑r
r =1 ( r + 1)( r + 2) r =1 ( r + 1)( r + 2) r =7

214 1 26 1 8
= − − + − (7 + 14)
14 + 2 2 6 + 2 2 2
= 1016 − 84
= 932

3. (2r – 1)2
= (4r2 – 4r + 1)
= 4r2 – 4r + 1
= (4n2 – 1) (given)
∴ 4r2 = (4n2 – 1) + 4r – 1
= (4n2 – 1) + 4 (n + 1) – n
= [ 4n2 – 1 + 6(n + 1) – 3 ]
= ( 4n2 + 6n + 2 )
= ( 2n2 + 3n + 1 )
∴ r = (2n + 1)(n + 1) (Shown)
2

32 + 62 + 92 + 122 + … + 992
= 3 2 + 3 2 2 2 + 3 2 3 2 + 3 2 4 2 + ... + 3 2 33 2
= 32 (12 + 22 + 32 + ... + 332)
33
=9 ∑r
r =1
2

= 9 [(66 + 1)(33 + 1)] (from above result)


= 112761

4. (i) β n − βn +1 = ( n + k ) α n −( n +1 +k) αn+1


n! ( n + 1) !
= ( n + k) − ( n + k +1)
( n + k) ! ( n + k +1) !
( n + k ) n!− ( n + 1) !
=
( n + k)!
n !( n + k − n − 1)
=
( n + k)!
n!
= ( k − 1) = ( k −1) α n (Shown)
( n + k)!
N
1 N
(ii) ∑α
n =1
n = ∑ ( k − 1) α n
k − 1 n =1
1 N
= ∑ ( β n − β n+1 )
k − 1 n =1

7
 β1 − β 2 
 
 + β2 − β3 
1  
= + M
k −1 
 + β N −1 − β N 
+ β − β 
 N N +1 

1
= ( β1 − β N +1 )
k −1
1  1 ( N + 1) ! 
= ( 1 + k ) − ( N + 1+ k ) 
k −1  ( 1+ k ) ! ( N + 1+ k ) !
1  1 ( N + 1) ! 
=  − 
k − 1  k ! ( N + k ) !

∞ N
(iii) ∑ α n = lim ∑ α n
n =1
N →∞
n =1

 1  1 ( N + 1) !  
= lim   − 
N →∞ k − 1 k !
  ( N + k ) ! 
 1  1 1  
= lim   − 
N →∞ k − 1 k !
  ( N + 2 ) ( N + 3) ... ( N + k )  
1 1 
=  − 0
k −1 k ! 
1
=
( k − 1) k !
5(i f ( x ) = 4 x − x2
)
Method 1 Method 2
Give a counter example
f ( 1) = 4 × 1 − 12 = 3
f ( 3) = 4 × 3 − 32 = 3
Since f ( 1) = f ( 3) = 3, f is not one-
one.
The graph of f fails the horizontal
line test, f is not one-one.
(ii)
(
f ( x ) = − x2 − 4x )
= −  x 2 − 2 ( x ) ( 2 ) + 22 − 22 

= − ( x − 2 ) − 4 
2

 
= 4 − ( x − 2)
2

The maximum point of the curve is ( 2, 4 ) . A = 2 for f −1 to exist

8
Therefore range of f is (−∞, 4] .
(iii Let 4 x − x = y = f ( x) , x ∈ D f ⇒ x ≤ 2
2

)
4 ± 16 − 4(1)( y )
x2 − 4 x + y = 0 ⇒ x =
2
4 ± 16 − 4 y
x=
2
x = 2± 4− y

Since x ≤ 2 , x = 2 − 4 − y
D f −1 = R f = (−∞, 4]
Therefore f ( x ) = 2 − 4 − x , x ≤ 4
−1

(iv
)

The graph of f and f −1 is the reflection of each other about the line y = x .

6. Time taken on nth day = 45(0.985)n −1 .

45(0.985)n −1 < 30
30
⇒ (n − 1) ln(0.985) < ln
45
30
⇒ n − 1 > ln ÷ ln(0.985)
45
⇒ n > 27.8
∴ least number of days =28.

From (i), Tony has to train daily for 28 days to obtain the ‘Running’
award.

9
Total time spent running over 28 days
28
= ∑ 45(0.985) n −1
n =1

45(1 − 0.98528 )
=
1 − 0.985
= 1035.12 mins (>15 hrs = 900 mins)

Thus, Tony is putting himself at risk of having asthma.

Statistics

7. (a)(i) No of arrangement = 4! = 24
(a)(ii)
Method 1
We find the sum of these 2 cases :
No of arrangement with first and last letter both I
6!
= 360
= 2! .
No of arrangement with at least one of first or last letter I
 7! 
  2 = 2520
=  2!2!  .
Total no of arrangement = 2520 − 360 = 2160 .

Method 2
We find the sum of these 3 cases :
No of arrangement with first and last letter both I
6!
= 360
= 2! .
No of arrangement with I and N at first and last letter respectively, and vice
versa
 6! 
2   = 720
=  2!  .
No of arrangement with exactly I at first letter, and neither N nor I at last
letter, and vice versa
 6!3 
2  = 1080
=  2!2!  .
Total no of arrangement = 2160.

(b) No of ways to arrange the other 4 students = 4!


No of ways to slot in 4 particular students = (5)(4)(3)(2)
Required no of ways = 4! (5)(4)(3)(2) = 2880

4
No of arrangement for 4 particular students = 3! 2
No of arrangement for other 4 students = 4!
4
Required no of arrangement = 3! 2 4! = 2304

10
8. (a) P(Tom wins) = P(drawing 2 red balls)
20 19 19
= ⋅ = = 0.244
40 39 78

(b) P(Jerry wins)


= P(Tom draws 2 green balls and Jerry draws 2 red balls) +
P(Tom draws 1 red, 1 green and Jerry draws 2 red balls)
20 19 20 19  20 20  19 18
= ⋅ . ⋅ + 2 ⋅ . ⋅
40 39 38 37  40 39  38 37
275
= = 0.191
1443

(c) P(Jerry wins\Tom draws a red ball and a green ball )


Jerry wins and Tom draws a red ball and a green ball
= P( )
Tom draws a red ball and a green ball
 20 20  19 18
2 ⋅ ⋅ ⋅
= 
40 39  38 37
 20 20 
2 ⋅ 
 40 39 
19 18 9
= ⋅ = = 0.243
38 37 37

P(Tom and Jerry each wins 2 games, given that all 4 games are won)
Tom wins 2 games and Jerry wins 2 games and 4 games are won
=P( )
4 games are won
2 2
 19   275  4!
   
 78   1443  2!2! = 0.364
= 4
 19 275 
 + 
 78 1443 

9. Let A be r.v. “weight of an apple in g”. A ~ N (µ, σ 2 )


P ( X > 200) = 0.1
200 − µ
P( Z > ) = 0.1
σ
200 − µ
P( Z < ) = 0.9
σ
200 − µ
= 1.2816 ----------(1)
σ

11
P ( X < 180) = 0.15
180 − µ
P( Z < ) = 0.15
σ
180 − µ
= −1.0364 ----------(2)
σ
Solving , µ = 188.94 and σ = 8.63 (shown) (ans)

∴ A ~ N (188.94,8.632 )
Let O be r.v. “weight of an orange in g”. O ~ N (220,9)
D = (O − A) , D ~ N (31.06,83.48)
P ( O − A < 10)
= P ( −10 < (O − A) < 10)
= 0.0106 (ans)

T = ( A1 + A2 + A3 ) + (O1 + O2 ) + 52
T ~ N (1058.82, 241.43)
P (T > 1030) = 0.968

10 (i) Poisson distribution is suitable as


. - No. of faults occurring increases proportionally to the duration of
time considered
- The rate of occurrence of faults is constant throughout the time
interval
- Faults occur singly and randomly

(ii) Let A and B denote the no. of V-22 and F-16 accidents occurring per
20,000 hours respectively.
A : Po(1.2), B : Po(0.8)
X = A + B : Po(2)
P ( A ≥ 2 & B ≥ 1| A + B = 4)
P ( A = 2 & B = 2) + P( A = 3 & B = 1)
=
P ( X = 4)
P ( A = 2) P ( B = 2) + P( A = 3) P( B = 1)
=
P ( X = 4)
0.062362
=
0.090223
= 0.691 (3 s.f.)
(iii) Let X denote no. of accidents occurring per 20,000 hrs.
X : Po(2) / 20, 000hrs
P ( X > n) ≤ 0.15
P ( X ≤ n) ≥ 0.85
By GC, n = 2 : P( X ≤ 2) = 0.677
n = 3 : P ( X ≤ 3) = 0.857 ≥ 0.85
∴ least n = 3

12
11 Let X denote the random variable representing the number of pieces of
(i) Maltz, out of 16, that are of poor quality.
X : B ( 16, 0.05)
P ( X ≥ 2) = 1 − P( X ≤ 1)
= 0.189

(ii) P ( exactly 2 of poor quality packet rejected)

C2 ( 0.05 ) ( 0.95 ) 14
16 2
P ( X = 2)
= =
P ( X ≥ 2) 0.189
= 0.774

(iii Let Y denote the random variable representing the number of packets of
) Maltz, out of 50, that are rejected.
Y : B ( 50, 0.189)
Since n = 50 is sufficiently large such that
np = 50 × 0.189 = 9.45 > 5 and n(1 − p ) = 50 × ( 1 − 0.189) = 40.55 > 5
we can approximate with the Normal distribution.
Hence, Y : N ( 9.45, 7.66395) approximately.
P ( Y ≤ 10 ) = P ( Y ≤ 10.5) by continuity correction
= 0.648

12 Let X be the amount received by a PA recipient.


. H 0 : µ = 260
H1 : µ > 260
Assuming X is from a normal population.
Under H T ~ t(14)
0
Using T-Test, with n = 15
X − µ 0 X − 260
t= =
s s
n 15
GC screen :

The amount of money received is enough ⇒ we do not reject H 0


α
So p = 0.224 > ⇒ α < 22.4 .
100
Hence, the amount of money is enough if we test at 22.4 % level of
significance.

13
Assuming X is from a normal population and sample size is large, she will
conduct a Z-test:
Test Statistic:
X − 260
z= ~N
s (0, 1) under H 0
n
For her evidence to be credible at 1% level of significance, reject H0
___
X − 260
> 2.326
s
n
263.66667 − 260
⇒ > 2.326
18.21171
n
⇒ n > 133.48

Least number of recipients to be interviewed = 134.

Since y = 1.0031x − 4.3018 , then we have


b = 1.0031 and y − bx = −4.3018 .

Hence y − 1.0031( 52.75 ) = −4.3018 ⇒ y = 48.611725

∑ y = 48.611725
8
338.7 + k
⇒ = 48.611725
8
⇒ k = 50.2 (ans)

13 From GC, we have r = 0.863 (ans)


.
Model C is suitable since it is observed from the scatter plot diagram that as
x increases, the rate at which y increases is decreasing. (ans)

Model C : y = a + b ln x
L1 → x
L2 → y
L3 → ln(L1 )
Using GC, LinReg L3 , L 2 yields y = 53.72ln x − 163.14
Hence a = −163.14, b = 53.72, r = 0.905 (ans)

80 = 53.72 ln x −163.14 ⇒ x = 92.39kg (ans)

Since y = 80 is out of the range, the value of x obtained may not be


accurate.

14
The End

15

Anda mungkin juga menyukai